KROK 2 DENTISTRY Flashcards

1
Q

A 30-year-old patient complains of a toothache caused by hot and cold stimuli. The pain irradiates to the ear and temple. Previously this tooth presented with spontaneous nocturnal toothache. Objectively: on the occlusal surface of the 37 tooth there is a deep carious cavity communicating at one point with the tooth cavity. Probing at the communication point, as well as cold stimulus, causes acute pain. The pain persists for a long time. Electric pulp test result is 5 microamperes. What is the most likely diagnosis?
Question 1Select one:

A.
Acute diffuse pulpitis

B.
Chronic concrementous pulpitis

C.
Exacerbation of chronic periodontitis

D.
Exacerbation of chronic pulpitis

E.
Acute suppurative pulpitis

A

D.
Exacerbation of chronic pulpitis

How well did you know this?
1
Not at all
2
3
4
5
Perfectly
2
Q

A 59-year-old man complains of painful edema that appeared 5 days ago in the right buccal region, an increase in body temperature to 37.5–38.0°C, and general weakness. Objectively, his face is asymmetrical due to painful swelling in the right parotid region, the skin over which is mildly hyperemic. Thick saliva with a small amount of purulent exudate is being produced from the excretory duct of the salivary gland. Make the provisional diagnosis.
Question 2Answer

a.
Abscess of the right parotid region

b.
Acute purulent parotitis

c.
Chronic interstitial parotitis

d.
Epidemic parotitis

e.
Herzenberg’s pseudoparotitis

A

B-Acute purulent parotitis

acute bacterial infection of the parotid gland, which is commonly referr

How well did you know this?
1
Not at all
2
3
4
5
Perfectly
3
Q

A 13-year-old girl complains of a sharp pain when biting on her tooth 21. The pain developed after an injury. Objectively, the crown of tooth 21 has two fracture lines, the fragments are slightly displaced, blood is oozing from the pulp chamber, percussion is painful. X-ray shows a diagonal fracture of the crown and root of tooth 21. The depth of the subgingival edge of the fracture line is over 4 mm. What should be the dentist’s tactics regarding tooth 21?
Question 3Answer

a.
Replantation

b.
Splinting

c.
Filling

d.
Extraction

e.
Monitoring for the next month

A 13-year-old girl complains of a sharp pain when biting on her tooth 21

a. Replantation: This option involves repositioning and securing the tooth in its socket. However, the depth of the subgingival edge of the fracture line, along with displacement and other complications, makes successful replantation less likely.

b. Splinting: While splinting is a technique used to stabilize teeth, it might not be effective in cases of severe fracture with significant displacement. The prognosis for splinting success depends on the stability of the fractured segments.

c. Filling: Filling is not suitable for a fractured tooth with involvement of the pulp chamber. It does not address the structural damage or provide a solution for stabilization.

e. Monitoring for the next month: Given the severity of the fracture, monitoring without intervention may lead to complications such as infection, pain, or damage to adjacent teeth.

A

d.
Extraction

The depth of the subgingival edge of the fracture line being over 4 mm, along with the displacement of fragments, suggests a severe and complicated fracture. In such cases, extraction is often the recommended approach, as attempts at replantation may not be successful.

How well did you know this?
1
Not at all
2
3
4
5
Perfectly
4
Q

Due to lack of timely specialized treatment a 44-year-old man presents with incorrectly healed displaced mandibular fracture. Objectively the lower jaw narrows sharply, vestibular cusps of the lower teeth contact with oral cusps of the upper teeth. The patient declined surgical treatment. What treatment tactics should be chosen by the dentist in this case?
Question 4Select one:

A.
Make a non-removable dental bridge with movable joint

B.
Make a prosthesis with double dentition

C.
Make a dentogingival laminar denture

D.
Correct the malocclusion by filing down the patient’s teeth

E.
Correct the malocclusion via instrumental surgical method

Due to lack of timely specialized treatment a 44-year-old man presents with incorrectly healed displaced mandibular fracture. Objectively the lower jaw narrows sharply, vestibular cusps of the lower teeth contact with oral cusps of the upper teeth. The patient declined surgical treatment. What treatment tactics should be chosen by the dentist in this case?
Question 4Select one:

A.
Make a non-removable dental bridge with movable joint

B.
Make a prosthesis with double dentition

C.
Make a dentogingival laminar denture

D.
Correct the malocclusion by filing down the patient’s teeth

E.
Correct the malocclusion via instrumental surgical method

A

B.
Make a prosthesis with double dentition

Creating a prosthesis with d

B. Make a prosthesis with double dentition

Creating a prosthesis with double dentition can help restore function and aesthetics by compensating for the malocclusion resulting from the incorrectly healed fracture. This option aims to provide the patient with a functional and esthetically pleasing solution without the need for surgical correction.
A. Make a non-removable dental bridge with movable joint: Creating a dental bridge may not be the ideal solution for an incorrectly healed displaced mandibular fracture. The movable joint might not effectively address the malocclusion and jaw narrowing.

D. Correct the malocclusion by filing down the patient’s teeth: Filing down the teeth (occlusal adjustment) might be considered, but it may not provide a comprehensive solution for the malocclusion caused by the incorrectly healed fracture. It may also lead to further complications.

E. Correct the malocclusion via instrumental surgical method: The patient has declined surgical treatment, so this option is not feasible based on the patient’s preferences.

Choosing option B, making a prosthesis with double dentition, is a conservative approach that addresses both the functional and aesthetic aspects without the need for surgical intervention, making it a more suitable choice in this specific scenario.

C. Make a dentogingival laminar denture: This option involves creating a denture that not only replaces missing teeth but also incorporates gingival elements for a more natural appearance. While it can address the aesthetic aspect of the malocclusion, it might not provide as much functional improvement as a prosthesis with double dentition.

How well did you know this?
1
Not at all
2
3
4
5
Perfectly
5
Q

A 36-year-old man complains of an ulcer on his lower left lip. Three weeks ago a smal round red spot appeared on his lip. Eventually it became more dense and noticeable and 2-3 days ago developed an ulcer in its center, while under the lower jaw appeared multiple painless nodules. Objectively on the mucosa of the lower left lip there is an oval ulcer, 1.2 cm in diameter, with smooth clear margins and bright-red glossy floor. Ulcer edges form a cushion that smoothly descends to the bottom of the ulcer. In the base of the ulcer there is a dense painless infiltration. Submandibular lymph nodes are enlarged and painless, the skin over them remains unchanged. This clinical presentation corresponds with the following disease:
Question 5Select one:

A.
Trophic ulcer

B.
Syphilis (initial period)

C.
Ulcerative cheilitis

D.
Lupus erythematosus

E.
Lip cancer

A

B.
Syphilis (initial period)

Round red spot evolving into an ulcer: This matches the typical progression of syphilitic lesions.

Painless nodules under the lower jaw (submandibular lymph nodes): Enlarged, painless lymph nodes are characteristic of syphilis.

Ulcer characteristics: Smooth clear margins, bright-red glossy floor, and the cushion-like descent to the bottom are consistent with syphilitic ulcers.

Now, let’s discuss why the other options are less likely:

A. Trophic ulcer: This typically refers to an ulcer resulting from a decrease in nerve supply or blood circulation. The described clinical presentation, along with the associated symptoms, is more suggestive of an infectious process like syphilis.

D. Lupus erythematosus: While lupus can affect the skin and mucous membranes, the specific characteristics of the ulcer and the presence of painless nodules under the jaw are more indicative of syphilis.

E. Lip cancer: Lip cancer could present with an ulcer, but the specific features, including the bright-red glossy floor and the painless nodules, make syphilis a more likely diagnosis in this context.

How well did you know this?
1
Not at all
2
3
4
5
Perfectly
6
Q

The department of dentofacial surgery admitted a patient who needs repair of a post-traumatic nose wing defect up to 3,0 cm in diameter. The trauma occured six months ago. What kind of grafting is indicated in this clinical situation?
Question 6Select one:

A.
Free grafting with dermal flap

B.
Grafting with local tissues of nasolabial or cheek regions

The department of dentofacial surgery admitted a patient who needs repair of a post-traumatic nose wing defect up to 3,0 cm in diameter. The trauma occured six months ago. What kind of grafting is indicated in this clinical situation?
Question 6Select one:

A.
Free grafting with dermal flap

B.
Grafting with local tissues of nasolabial or cheek regions

C.
Grafting with chondrocutaneous flap of the auricle

D.
Grafting with tubed pedicle flap (Filatov’s flap)

E.
Grafting with pedicle flap of frontal and buccal regions

D.
Grafting with tubed pedicle flap (Filatov’s flap)

E.
Grafting with pedicle flap of frontal and buccal regions

A

C.
Grafting with chondrocutaneous flap of the auricle

How well did you know this?
1
Not at all
2
3
4
5
Perfectly
7
Q

A 15-year-old girl complains of throbbing pain and a swelling above her upper lip, fever of 38.0°С, and deterioration of her general condition. Two days ago she tried to pop a pimple in this area on her own. Objectively, she has a dense painful infiltration on the skin of her upper lip. The infiltration is rounded, cone-shaped, up to 2.5 cm in diameter. The skin over the infiltration is sharply hyperemic, with a necrotic rod in the center. Make the diagnosis.
Question 7Answer

a.
Hematoma of the upper lip at the stage of suppuration

b.
Atheroma of the upper lip at the stage of suppuration

c.
Carbuncle on the upper lip

d.
Odontogenic abscess of the upper lip

e.
Furuncle with abscess on the upper lip

A

e.
Furuncle with abscess on the upper lipn

Furuncle on the upper lip (Option e):

The description of a dense, painful infiltration with hyperemic skin, throbbing pain, and a necrotic center is typical of a furuncle.
Furuncles are localized infections of a hair follicle and surrounding tissue, often caused by Staphylococcus aureus.
Why not the other options:

Hematoma of the upper lip at the stage of suppuration (Option a): Hematomas are collections of blood, and the description doesn’t align with the typical progression of a hematoma.

Atheroma of the upper lip at the stage of suppuration (Option b): Atheromas are cystic structures involving sebaceous glands, and they usually don’t present with the described symptoms of hyperemia and necrosis.

Carbuncle on the upper lip (Option c): A carbuncle involves multiple hair follicles and is characterized by interconnected abscesses. The described cone-shaped infiltration is more typical of a furuncle.

Odontogenic abscess of the upper lip (Option d): While dental infections can cause facial swelling, the acute onset, throbbing pain, and skin findings are more indicative of a furuncle.

How well did you know this?
1
Not at all
2
3
4
5
Perfectly
8
Q

A 55-year-old woman needs removable implant-retained dentures for both jaws. It is necessary to choose the type, the number and the placement of implants. What additional examination method will be the most efficient in making this decision?
Question 8Select one:

A.
Spot X-ray

B.
Occlusography

C.
Computed tomography

D.
Study of diagnostic models

E.
Ultraviolet densitometry

A

C.
Computed tomography

How well did you know this?
1
Not at all
2
3
4
5
Perfectly
9
Q

A 47-year-old patient presents with rounded bone protrusions 0,7-0,8 cm in size on the inner surface of the edentulous mandible in the premolar area. The denture for this patient should have:
Question 9Select one:

A.
Dentogingival clasps

B.
Elastic liner

C.
Orifices for the exostoses

D.
Kemeny clasps

E.
Metal base

A

B.
Elastic liner

An elastic liner can help provide a cushioning effect and enhance the denture’s adaptation to the rounded bone protrusions on the inner surface of the edentulous mandible. This option aims to improve comfort and reduce potential irritation caused by the bony irregularities during denture use.

How well did you know this?
1
Not at all
2
3
4
5
Perfectly
10
Q

The medical station of a regiment received a patient with signs of bilateral mandibular fracture. What is the main task of first aid in this case?
Question 10Select one:

A.
To provide symptomatic therapy and care

B.
To control shock, bleeding, and asphyxia and to provide transport immobilization

C.
To check and correct previously applied bandages

D.
To clean the oral cavity from blood clots, tooth shards, and bone fragments

E.
To administer analgesics and cardiac medications

A

B.
To control shock, bleeding, and asphyxia and to provide transport immobilization

How well did you know this?
1
Not at all
2
3
4
5
Perfectly
11
Q

A 10,5-year-old child complains of painful rash on his lips. Objectively: the red border of the lips is swollen, hyperemic, covered in fissures and numerous scabs of dried blood. The skin of the upper lip has small blisters containing serous substance, which merge with each other in some places. Maceration and weeping skin also can be observed, especially in the corners of the mouth. What is the most likely diagnosis?
Question 11Select one:

A.
Atopic cheilitis

B.
Exfoliative cheilitis

C.
Actinic cheilitis

D.
Exematous cheilitis

E.
Meteorological cheilitis

A

D.
Exematous cheilitis

The presence of a painful rash on the lips, swollen and hyperemic red border, fissures, scabs of dried blood, small blisters containing serous substance, and maceration with weeping skin, especially in the corners of the mouth, is consistent with exematous cheilitis. This type of cheilitis is often associated with inflammation and can manifest in children.

User
why not otherss
ChatGPT
Let’s discuss why the other options may not be the most likely diagnosis:

A. Atopic cheilitis: Atopic cheilitis is associated with conditions like eczema and is characterized by dry, itchy, and inflamed lips. While some symptoms overlap, the presence of blisters containing serous substance and the description of weeping skin are more indicative of exematous cheilitis.

B. Exfoliative cheilitis: Exfoliative cheilitis typically presents with continuous peeling of the lips. The presence of blisters, dried blood, and weeping skin doesn’t align with the primary characteristics of exfoliative cheilitis.

C. Actinic cheilitis: Actinic cheilitis is often related to sun damage and presents with dry, scaly, or thickened lips. It is less likely given the description of small blisters, weeping skin, and the age of the patient.

E. Meteorological cheilitis: Meteorological cheilitis refers to lip dryness and cracking due to weather conditions. However, the presence of blisters, weeping skin, and scabs suggests a more inflammatory condition like exematous cheilitis

How well did you know this?
1
Not at all
2
3
4
5
Perfectly
12
Q

A 27-year-old woman complains of painful and bleeding gums, with the signs aggravating during eating, and indisposition. One week ago she had a case of URTI. Within the last 5 years she periodicaly presents with gingival hemorrhages. Objectively: the gums are bright red, markedly swollen; gingival papillae are friable, bleed on the slightest touch. There is moderate accumulation of dental calculus, large amount of soft dental deposit. Submaxillary lymph nodes are enlarged, painful on palpation. On X-ray: osteoporosis of alveolar septa is observed. What is the most likely diagnosis?
Question 12Select one:

A.
Exacerbation of chronic catarrhal gingivitis

B.
Acute catarrhal gingivitis

C.
Exacerbation of initial generalized periodontitis

D.
Hypertrophic gingivitis, edematous form

E.
Hypertrophic gingivitis, fibrous form

A

A.
Exacerbation of chronic catarrhal gingivitis

How well did you know this?
1
Not at all
2
3
4
5
Perfectly
13
Q

A 17-year-old patient has been suffering for 2 years from periodical painful swelling and fistula appearing on the skin of gonial angle area. After cleaning the fistula of its purulent content pain and swelling disappear leaving a small concave scar. Objectively: there is a small scar on the skin in the gonial angle area. Palpation of the oral cavity floor reveals a dense band in the buccal soft tissues, which connects fistula with projection of the 47 tooth roots. Crown of the 47 is destroyed. What diagnosis is most likely?
Question 13Select one:

A.
Chronic osteomyelitis of the left mandibular angle

B.
Mandibular tuberculosis

C.
Actinomycosis of the parotid-masticatory area

D.
Festered buccal atheroma

E.
Migrating subcutaneous granuloma
Feedback

A

E-Migrating subcutaneous granuloma
Feedback

How well did you know this?
1
Not at all
2
3
4
5
Perfectly
14
Q

A 64-year-old patient complains of spots on the mucosa of the cheeks, mouth angles, palate. Objectively: on the oral cavity mucosa there are white growths that cannot be removed with spatula. The mucosa around the spots is pale pink in color. The patient has been smoking for 40 years. What is the most likely diagnosis?
Question 14Select one:

A.
Candidiasis

B.
Bowen’s disease

C.
Leukoplakia

D.
Lupus erythematosus

E.
Lichen ruber planus

A

C.
Leukoplakia

The most likely diagnosis based on the described symptoms is:

C. Leukoplakia

The presence of white growths on the oral cavity mucosa that cannot be removed with a spatula, along with a history of smoking, is indicative of leukoplakia. Leukoplakia is a potentially precancerous condition associated with tobacco use.

It’s crucial for the patient to consult with a healthcare professional for a thorough examination and appropriate management, as leukoplakia may require further evaluation and monitoring due to its potential link to oral cancer.

User
why not other
ChatGPT
Let’s discuss why the other options are less likely based on the presented symptoms:

A. Candidiasis: Candidiasis often presents as white patches that can be scraped off, which is not the case here. Additionally, the patient’s long-term smoking history is more suggestive of leukoplakia.

B. Bowen’s disease: Bowen’s disease typically presents as a precancerous skin lesion, and its occurrence in the oral mucosa is uncommon. The described symptoms are not typical for Bowen’s disease.

D. Lupus erythematosus: Lupus erythematosus primarily affects the skin and can involve the oral mucosa, but the described white growths are not typical for this condition.

E. Lichen ruber planus: Lichen planus usually presents with lacy white lines (Wickham’s striae) and may involve the oral mucosa. However, the description of white growths that cannot be removed with a spatula is more indicative of leukoplakia.

How well did you know this?
1
Not at all
2
3
4
5
Perfectly
15
Q

A 44-year-old man complains of indisposition, fever of 38°С, and a swelling under the lower jaw on the left. The patient’s history states that throughout the last week he felt pain in the area of his tooth 36. Objectively, in the left submandibular area there is an enlarged painful nodule. The skin above the nodule is hyperemic, strained, and cannot be pinched. Tooth 36 is partially destroyed in the oral cavity. What diagnosis can be made in this case?
Question 15Answer

a.
Abscess of the submandibular region

b.
Acute serous lymphadenitis

c.
Acute purulent lymphadenitis

d.
Furuncle

e.
Chronic lymphadenitis

A

c.
Acute purulent lymphadenitis

The presence of an enlarged, painful nodule in the left submandibular area, along with hyperemic and strained skin, is indicative of acute inflammation and infection of the lymph nodes (lymphadenitis). The history of tooth pain and partial destruction of tooth 36 suggests a possible source of infection leading to purulent lymphadenitis.

How well did you know this?
1
Not at all
2
3
4
5
Perfectly
16
Q

A 22-year-old student complains of gingival hemorrhage during tooth brushing. Objectively: hyperemia and edema of marginal gingiva of the front teeth on both jaws, periodontal pockets are absent. X-ray reveals no pathological changes. What is the most likely diagnosis?
Question 16Select one:

A.
Chronic catarrhal gingivitis

B.
Generalized periodontitis

C.
Hypertrophic gingivitis

D.
Localized periodontitis

E.
Ulcerative gingivitis

A

A.
Chronic catarrhal gingivitis

How well did you know this?
1
Not at all
2
3
4
5
Perfectly
17
Q

A patient complains of burning in the area of the hard palate mucosa when using a partial removable laminar denture made one week ago. The following diagnosis has been made: allergic contact stomatitis. Determine the treatment tactics, if it is known that there are no conditions for making an arch denture and the patient is using a removable denture for the first time.
Question 17Answer

a.
Make a new removable denture with a metal base

b.
Boil the previously made denture in distilled water for 10 minutes

c.
Make a new denture, adding a smaller amount of monomer to the plastic dough

d.
Make a new denture, adding a smaller amount of polymer to the plastic dough

e.
Apply the previously made denture again with frequent rinsing with water and taking hyposensitizing agents

A

a.
Make a new removable denture with a metal base

This involves changing the material of the denture to one that the patient is not allergic to. Using a metal base can be a suitable alternative that reduces the risk of an allergic reaction. Additionally, ensuring proper fit and comfort is essential during the fabrication of the new denture.

How well did you know this?
1
Not at all
2
3
4
5
Perfectly
18
Q

One hour ago a patient received a blunt object trauma to the middle area of his face. X-ray revealed a fracture line in the bone tissue that passes through the nasal root, medial wall of the eyesocket, pterygoid processes of the sphenoid bone, and zygomatic arch. Make the diagnosis:
Question 18Select one:

A.
Le Fort II maxillary fracture

B.
Nasal and zygomatic bone fracture

C.
Le Fort III maxillary fracture

D.
Zygomatic bone fracture

E.
Le Fort I maxillary fracture

A

C
Le Fort III maxillary fracture

Considering the involvement of multiple structures, including the nasal root, medial wall of the eye socket, pterygoid processes of the sphenoid bone, and zygomatic arch, the most appropriate diagnosis is Le Fort III maxillary fracture. This type of fracture typically results from severe trauma to the midface and is characterized by extensive separation of the facial bones from the cranial base.

The other options are less suitable for this specific pattern of fractures:

Le Fort II maxillary fracture (Option A): Typically involves the nasal bone, maxilla, and infraorbital rim but not the pterygoid processes or the zygomatic arch.

Nasal and zygomatic bone fracture (Option B): This description doesn’t encompass the involvement of the pterygoid processes or the medial wall of the eye socket.

Zygomatic bone fracture (Option D): This wouldn’t account for the fractures through the nasal root, medial wall of the eye socket, or pterygoid processes.

Le Fort I maxillary fracture (Option E): Usually involves the maxilla but not to the extent described, including the fractures through the pterygoid processes and the zygomatic arch.

In summary, the combination of fractures described aligns most closely with Le Fort III maxillary fracture.

How well did you know this?
1
Not at all
2
3
4
5
Perfectly
19
Q

A 25-year-old HIV-infected patient came to a clinic of prosthetic dentistry to have a denture made for him. What aseptic and antiseptic precautions should be taken?
Question 19Select one:

A.
The orthopaedist should work in gloves and a mask

B.
The impressions should be desinfected under a quartz lamp

C.
No special precautions are required

D.
According to the scheme

E.
The patient should be refused appointment

A

D.
According to the scheme

How well did you know this?
1
Not at all
2
3
4
5
Perfectly
20
Q

A patient with post-resection upper jaw defect that invades the nasal cavity has come to a prosthodontic clinic. What denture is recommended in the given case?
Question 20Select one:

A.
Forming denture

B.
Floating obturator

C.
Replacement denture

D.
Mouthguard

E.
Replacement denture with obturating element

A

E.
Replacement denture with obturating element

How well did you know this?
1
Not at all
2
3
4
5
Perfectly
21
Q

A 28-year-old man came to a dentist with complaints of pain in his lower left tooth. The pain is provoked by sweet foods and stops immediately after the stimulus is removed. Objectively, the distal surface of tooth 34 has a deep carious cavity filled with soft pigmented dentin. Probing the of the cavity floor is painful. No communication can be detected between the cavity and the pulp chamber. Thermal stimulation results are positive, short-term. What diagnosis can be made in this case?
Question 21Answer

a.
Chronic fibrous periodontitis

b.
Acute deep caries

c.
Chronic gangrenous pulpitis

d.
Chronic deep caries

e.
Acute limited pulpitis

A

b.
Acute deep caries

How well did you know this?
1
Not at all
2
3
4
5
Perfectly
22
Q

A 26-year-old woman, who works with chemicals at a factory, came to a dentist with complaints of teeth discoloration and sharp sensitivity to cold, sour, and sweet in her front upper teeth. Objectively, on the vestibular surface of teeth 13, 12, 11, 21, 22, 23 at the level of the equator, there are white patches that spread to the cutting edges. During probing, the enamel in the affected area is rough and fragile. Vital staining is positive. Make the clinical diagnosis.
Question 22Answer

a.
Systemic enamel hypoplasia

b.
Erosion of dental hard tissues

c.
Acute initial caries

d.
Necrosis of dental hard tissues

e.
Dental hyperesthesia

A

d.
Necrosis of dental hard tissues

How well did you know this?
1
Not at all
2
3
4
5
Perfectly
23
Q

A 22-year-old patient came to a dentist with complaints of sharp pain in the oral cavity during eating and talking, increased salivation. The patient notes the acute onset of the disease after an overexposure to cold, high fever (39.5°C), general weakness, and headache. The patient has a history of chronic tonsillitis and allergy to some medicines. During examination, the following is observed: on the hyperemic and edematous mucosa of the lips and cheeks, there are numerous painful erosions covered with fibrinous coating; vermilion border of the lips is edematous and has blood scabs. The skin is pale, there are bluish-pink rashes with hemorrhagic scabs on the backs of the patient’s hands. Regional lymph nodes are enlarged, painful, and mobile. Make the provisional diagnosis.
Question 23Answer

a.
Acute herpetic stomatitis

b.
Stevens-Johnson syndrome

c.
Drug-induced stomatitis

d.
Erythema multiforme exudativum

e.
Acantholytic pemphigus

A

D
Erythema multiforme exudativum

How well did you know this?
1
Not at all
2
3
4
5
Perfectly
24
Q

A dental orthodontist has diagnosed an 11-year-old child with microdontia of the upper incisors. What diagnostic method was used in this case?
Question 24Answer

a.
Korkhauz

b.
Howes

c.
Pont

d.
Gerlach

e.
Tonn

A

e.
Tonn

How well did you know this?
1
Not at all
2
3
4
5
Perfectly
25
Q

When pressing plastic dough, the dental technician opened and closed the cuvette several times to control the amount of the added mass. What can develop as a result of such actions?
Question 25Answer

a.
Gas porosity

b.
Porosity caused by compression

c.
Cracks in plastic

d.
Residual stresses within the denture

e.
Granular porosity

A

e.
Granular porosity

How well did you know this?
1
Not at all
2
3
4
5
Perfectly
26
Q

A 24-year-old man was for the first time diagnosed with sialolithiasis with salivary stone localization in the duct of the submandibular salivary gland. What would be the optimal treatment tactics in this case?
Question 26Answer

a.
Anti-inflammatory pharmacotherapy

b.
Marsupialization surgery

c.
Sialolith removal

d.
Removal of the submandibular gland

e.
Lithotripsy

A

c.
Sialolith removal

How well did you know this?
1
Not at all
2
3
4
5
Perfectly
27
Q

A 50-year-old man was diagnosed with sialolithiasis with the salivary gland stone located deep within the salivary gland. Choose the optimal treatment tactics:
Question 27Select one:

A.
Sclerotherapy

B.
Radiation therapy

C.
Conservative pharmacotherapy

D.
Submandibular gland excision

E.
Removal of the sialolith while retaining the gland

A

D.
Submandibular gland excision

When the stone is deep within the salivary gland and causing recurrent or severe symptoms, surgical intervention such as submandibular gland excision may be necessary to address the condition effectively. This procedure involves the removal of the affected salivary gland.

How well did you know this?
1
Not at all
2
3
4
5
Perfectly
28
Q

A 35-year-old woman has complaints of cosmetic defects of the front upper teeth crowns. The defects have been aggravating for the last 10 years. The patient suffers from unpleasant sensations when brushing her teeth, and when chemical stimuli are applied. Objective examination revealed defects localized in the enamel of the front upper teeth vestibular surface. The defects are oval, saucer-shaped, and have clear margins. Response to probing and cold stimuli was positive. Make the diagnosis:
Question 28Select one:

A.
Cuneiform defect

B.
Hyperesthesia of tooth hard tissues

C.
Chemical necrosis of the tooth

D.
Enamel hypoplasia

E.
Enamel erosion

A

E.
Enamel erosion

How well did you know this?
1
Not at all
2
3
4
5
Perfectly
29
Q

Before performing a surgery a dental surgeon treats his hands according to Spasokukotsky-Kochergin method. What antiseptic is basic in this kind of treatment and how is it applied?
Question 29Select one:

A.
0,5% liquid ammonia solution in two washbasins, 3-5 minutes in each

B.
Two tissues with 96% ethanol, 5 minutes each

C.
Tissue with 98% ethanol, 5 minutes

D.
Two tissues with 5% liquid ammonia solution, 5 minutes each

E.
Tissue with 0,5% chlorhexidine digluconate, 5 minutes

A

A.
0,5% liquid ammonia solution in two washbasins, 3-5 minutes in each

How well did you know this?
1
Not at all
2
3
4
5
Perfectly
30
Q

A 25-year-old man was diagnosed with acute deep caries of tooth 13, Black class V. Photopolymer composite material is planned to be used for aesthetical restoration of tooth 13. What material should be used as a lining for the floor of the carious cavity in this case?
Question 30Select one:

A.
Calcium-containing paste

B.
Resorcin-formalin paste

C.
Dentin paste

D.
Iodoform paste

E.
Devitalizing paste

A

A.
Calcium-containing paste

Calcium-containing paste is often used as a liner in restorative dentistry to promote dentin remineralization and protect the pulp. It aids in the formation of tertiary dentin and acts as a protective barrier between the dental pulp and restorative materials.

How well did you know this?
1
Not at all
2
3
4
5
Perfectly
31
Q

An 11-year-old boy complains of a carious cavity in the mandibular tooth on the right. Objectively: the 46 tooth exhibits a carious cavity within the mantle dentin. The dentin is dense, pigmented; there is pain response to the cold stimulus; probing and percussion cause no pain response. Make the provisional diagnosis:
Question 31Select one:

A.
Acute deep caries

B.
Chronic deep caries

C.
Chronic median caries

D.
Acute median caries

E.
Chronic superficial caries

A

C.
Chronic median caries

How well did you know this?
1
Not at all
2
3
4
5
Perfectly
32
Q

During an appointment with the dentist, a 42-year-old man developed anaphylactic shock after he was given an analgesic. What emergency first-line drug should be administered in this case?
Question 32Select one:

A.
Oxygen

B.
Antihistamine

C.
Inhaled short acting beta-2-agonist

D.
Glucocorticosteroid

E.
Adrenaline (epinephrine)

A

E.
Adrenaline (epinephrine)

How well did you know this?
1
Not at all
2
3
4
5
Perfectly
33
Q

A 45-year-old man complains of pain and crepitation in the temporomandibular joint during the movements of the lower jaw. Objectively: the face is symmetrical, the mouth opens with slight displacement to the left. Dentition is intact. To clarify the diagnosis, X-ray of the temporomandibular joint was performed. Where should the heads of the mandible be located normally during maximum mouth opening?
Question 33Select one:

A.
In the center of the articular tubercle

B.
At the top of the articular tubercle

C.
In the center of the glenoid fossa

D.
Closer to the distal part of the glenoid fossa

E.
In front of the articular tubercle

A

B.
At the top of the articular tubercle

How well did you know this?
1
Not at all
2
3
4
5
Perfectly
34
Q

A 42-year-old man was delivered to the hospital in the severe condition: inert, body temperature is 39.1°C, there is acutely painful infiltration of the mouth floor and submandibular area on the right. The skin over the infiltration is turgid and cyanotic. Palpation detects crepitus under the skin. What diagnosis can be made in this case?
Question 34Select one:

A.
Ludwig’s angina (suppurative-necrotic phlegmon of the mouth floor)

B.
Odontogenic phlegmon of the mouth floor

C.
Actinomycosis of the mouth floor

D.
Malignant tumor of the mouth floor

E.
Adenophlegmon of the mouth floor

A

A.
Ludwig’s angina (suppurative-necrotic phlegmon of the mouth floor)

Ludwig’s angina is a severe and potentially life-threatening condition characterized by a rapidly spreading cellulitis involving the submandibular, sublingual, and submental spaces. The condition often results in swelling, pain, and crepitus in the mouth floor and submandibular area. The turgid and cyanotic skin over the infiltration, along with systemic symptoms like fever, are consistent with Ludwig’s angina. It requires prompt medical attention and intervention due to the risk of airway compromise.

How well did you know this?
1
Not at all
2
3
4
5
Perfectly
35
Q

A 7-year-old child is sick for 3 days already. The child has fever of 38°С, weakness, and facial asymmetry due to edema of the upper lip and left infraorbital area. The skin color is unchanged. The mucogingival junction in the projection of the roots of teeth 63 and 64 is smoothed-out and hyperemic. Palpation detects a positive sign of fluctuation. Tooth 63 is partially destroyed, mobile, its percussion is slightly painful. Make the provisional diagnosis.
Question 35Answer

a.
Acute odontogenic osteomyelitis of the upper jaw on the left

b.
Acute serous periodontitis of tooth 63

c.
Acute purulent odontogenic periostitis of the upper jaw on the left

d.
Acute purulent odontogenic lymphadenitis

e.
Phlegmon of the infraorbital space on the left

A

c.
Acute purulent odontogenic periostitis of the upper jaw on the left

Odontogenic periostitis refers to inflammation of the periosteum, which is the dense connective tissue covering the outer surface of bones. In the context of dental pathology, odontogenic periostitis commonly arises from infections originating in the teeth or adjacent structures. The inflammation can spread to involve the periosteum, leading to clinical signs such as facial swelling, hyperemia, and tenderness.

In the case described with the 7-year-old child, acute purulent odontogenic periostitis is suspected due to the presentation of facial asymmetry, edema of the upper lip and left infraorbital area, hyperemia, and dental symptoms such as a partially destroyed and mobile tooth with a positive fluctuation sign. This condition typically requires prompt dental and medical intervention to address the underlying infection and prevent further complications.

If you have any more questions or if there’s anything else I can help you with, feel free to let me know!

How well did you know this?
1
Not at all
2
3
4
5
Perfectly
36
Q

A 2.5-year-old child is registered for regular check-ups with the orthodontist. The I stage of physiogical occlusion development corresponds with eruption of the following group of temporary teeth:
Question 36Select one:

A.
Temporary lateral incisors

B.
Temporary canines

C.
Front teeth

D.
Temporary molars

E.
Temporary central incisors

A

D.
Temporary molars

How well did you know this?
1
Not at all
2
3
4
5
Perfectly
37
Q

A 68-year-old patient addressed a surgical department of a dental clinic for extraction of the 45 tooth. During procedure the patient developed burning retrosternal pain attack irradiating to the left shoulder, scapula, hand. The skin is pale, BP is 140/100 mm Hg, heart rate is rapid. Skin hyperplasia can be observed in the Zakharin-Head’s zones. What emergency condition did the patient develop?
Question 37Select one:

A.
Bronchial asthma attack

C.
Heart failure

D.
Angina pectoris attack

E.
Hypertensic crisis

A

D.
Angina pectoris attack

How well did you know this?
1
Not at all
2
3
4
5
Perfectly
38
Q

A 25-year-old man came to a dentist with complaints of a cavity in his left lower tooth and pain during eating sweet, sour, and solid foods. Objectively, tooth 36 has a deep carious cavity filled with light-colored softened dentin. Probing of the cavity floor is painful, a cold stimulus provokes pain response that stops immediately after the stimulus is removed. What is the most likely diagnosis in this case?
Question 38Answer

a.
Chronic deep caries

b.
Acute deep caries

c.
Acute focal pulpitis

d.
Acute median caries

e.
Chronic fibrous pulpitis

A

b.
Acute deep caries

How well did you know this?
1
Not at all
2
3
4
5
Perfectly
39
Q

he parents of an 8-year-old girl came to a dental orthodontist complaining of an aesthetic defect in their child. Objectively, the lower part of the face is shortened, the chin is pushed forward, the upper lip sinks. When teeth are closed, reverse deep incisal overlap is revealed. Mesioocclusion is observed laterally. Select the appliance for the treatment.
Question 39Answer

a.
Osadchy appliance

b.
Frankel functional regulator, type 1

c.
Frankel functional regulator, type 3

d.
Andresen-Haupl activator

e.
Frankel functional regulator, type 2

A

c.
Frankel functional regulator, type 3

How well did you know this?
1
Not at all
2
3
4
5
Perfectly
40
Q

A 32-year-old man has been hospitalized into the maxillofacial department of a clinic. The following diagnosis has been made: an open displaced bilateral fracture of the lower jaw in the area of the gonial angle. X-ray shows a large diastasis between the fragments and muscle interposition. What main treatment technique should be chosen for this patient?
Question 40Answer

a.
Make a Vankevich splint for the fixation of the lower jaw

b.
Perform bilateral osteosynthesis of the lower jaw

c.
Make a Port splint for the fixation of the lower jaw

d.
ply a splint with loops for the lower jaw

e.
Apply a smooth splint-bracket

A

b.
Perform bilateral osteosynthesis of the lower jaw

Bilateral osteosynthesis involves surgically realigning the fractured fragments and securing them in place with fixation devices such as plates and screws. This approach is commonly used for displaced fractures to ensure proper alignment and stability during the healing process.

If you have any more questions or need further clarification, feel free to ask!

How well did you know this?
1
Not at all
2
3
4
5
Perfectly
41
Q

A group of experts conducts epidemiologic investigation of certain age groups by severity of signs of periodontal damage and their need for treatment. What index allows to study these criteria?
Question 41Select one:

A.
PІ (Russel)

B.
PDІ (Ramfjord)

C.
PMA (Parma)

D.
CPITN (WHO)

E.
OHІ-S (Green-Vermillion)

A

D.
CPITN (WHO)

How well did you know this?
1
Not at all
2
3
4
5
Perfectly
42
Q

A 21-year-old woman after a domestic accident presents with a fracture of the maxillary alveolar process in the area of teeth 13, 12, 11, 21, 22, and 23. What splint should be used for this fracture?
Question 42Select one:

A.
Smooth dental brace

B.
With retention loops

C.
With supporting plane

D.
With inclined plane

E.
With spreader arch

A

A.
Smooth dental brace

How well did you know this?
1
Not at all
2
3
4
5
Perfectly
43
Q

A 24-year-old woman is at the III trimester of her pregnancy. She complains of white spots appearing on her lower front teeth. Examination detects chalk-like lesions on the vestibular surfaces of teeth 41, 42, 31, 32 in the precervical area. The lesions can be stained with dyes. Choose the treatment tactics:
Question 43Select one:

A.
Preparation and filling

B.
Preparation without filling

C.
ART-technique

D.
Remineralization therapy

E.
Impregnation with silver nitrate

A

D.
Remineralization therapy

How well did you know this?
1
Not at all
2
3
4
5
Perfectly
44
Q

An 18-year-old student needs prosthetic porcelain-fused-to-metal denture for the 11, 21 teeth. There are no contraindications for the use of such construction. What is the most appropriate material for taking impressions?
Question 44Select one:

A.
Sielast

B.
Stomalgin

C.
Repin

D.
Orthocor

E.
Stens

A

A.
Sielast

Sielast is a commonly used elastomeric impression material in dentistry. It provides good detail reproduction and is suitable for various prosthetic applications, including the fabrication of porcelain-fused-to-metal dentures.

How well did you know this?
1
Not at all
2
3
4
5
Perfectly
45
Q

The maxillofacial surgery unit received a patient with complaints of inability to close his mouth. This condition occurred when the patient was biting an apple. Objectively there is a frightened expression on the patient’s face, the mouth is open wide, the chin is displaced to the left, salivation is observed. Palpation through the external acoustic meatus detected no movements of the right articular head. What is the most likely diagnosis?
Question 45Select one:

A.
Temporomandibular joint pain dysfunction syndrome

B.
Right temporomandibular joint dislocation

C.
Acute temporomandibular arthritis

D.
Fracture of the mandibular process

E.
Bilateral temporomandibular joint dislocation

A

B.
Right temporomandibular joint dislocation

How well did you know this?
1
Not at all
2
3
4
5
Perfectly
46
Q

A 47-year-old woman complains of an itching sensation in her gums and increased teeth sensitivity to thermal stimuli. Objectively, the roots of most of her teeth are exposed to 1/3 of their length, the gums are pale pink, no periodontal pockets. X-ray shows osteosclerosis of the alveolar process bone tissue, disturbed intactness of the lamina dura on the apices of the interdental septa. The patient was diagnosed with initial periodontosis. What should be prescribed in this case for local treatment of the systemic dental hyperesthesia?
Question 46Select one:

A.
Desensitizers

B.
Keratoplastics

C.
Keratolytics

D.
Anesthetics

E.
Antiseptics

A

A.
Desensitizers

How well did you know this?
1
Not at all
2
3
4
5
Perfectly
47
Q

A 16-year-old girl suffers from enlarged gums. Objectively: gingival margin of the vestibular surface of the gums is diffusely thickened, dense, pale pink in color. Gingival papilla cover crowns of all the teeth up to 1/2 of their height, hemorrhage is absent. X-ray reveals no pathological changes of periodontal tissues. Make the diagnosis:
Question 47Select one:

A.
Gingival fibromatosis

B.
Chronic catarrhal gingivitis

C.
Exacerbation of catarrhal gingivitis

D.
Chronic hypertrophic gingivitis

E.
Acute catarrhal gingivitis

A

D.
Chronic hypertrophic gingivitis

How well did you know this?
1
Not at all
2
3
4
5
Perfectly
48
Q

A 30-year-old woman addressed a dental surgeon with complaints of uncomfortable sensation of pressure in her upper right tooth, which aggravates due to hot stimulus, and foul smell from the mouth. Objectively: there is a deep carious cavity in the 17 tooth, which penetrates into the tooth cavity. Deep probing causes severe pain, percussion of the 17 tooth is painful. X-ray: there is slight widening of the periodontal fissure near the root apex. Electric pulp test - 70 microamperes. What final diagnosis can be made?
Question 48Select one:

A.
Chronic fibrous periodontitis

B.
Exacerbation of chronic fibrous periodontitis

C.
Acute purulent pulpitis

D.
Chronic fibrous pulpitis

E.
Chronic gangrenous pulpitis

A

E.
Chronic gangrenous pulpitis

How well did you know this?
1
Not at all
2
3
4
5
Perfectly
49
Q

An 11-year-old child presents with protrusion **of the upper front teeth, there are tremata and diastema between the child’s teeth. What apparatus can be used for treatment of this pathology?
Question 49Select one:

A.
Frankel, type 3

B.
Mershon

C.
Vasylenko

D.
Osadchy

E.
Brukle

A

D.
Osadchy

How well did you know this?
1
Not at all
2
3
4
5
Perfectly
50
Q

A 28-year-old man came to the maxillofacial surgery department. He complains of a cutaneous neoplasm in his right submandibular region. The neoplasm appeared approximately 2 years ago. In the process of shaving, the neoplasm was frequently injured. He was diagnosed with papilloma. In the neoplastic area, examination shows an ulcer against the background of hyperemic skin. What tactics should the dental surgeon choose?
Question 50Select one:

A.
Antibacterial and anti-inflammatory therapy, observation

B.
Removal of the papilloma and postoperative radiation therapy

C.
Anti-inflammatory therapy followed by removal of the papilloma

D.
Anti-inflammatory therapy followed by observation

E.
Prevention of malignant transformation of the papilloma. Removal of the papilloma

A

E.
Prevention of malignant transformation of the papilloma. Removal of the papilloma

How well did you know this?
1
Not at all
2
3
4
5
Perfectly
51
Q

A 21-year-old woman complains of short-term pain attacks in her lower left tooth that are provoked by thermal stimuli and quickly stop after the stimulus is removed. On the masticatory surface of tooth 47, examination reveals a carious cavity with a narrow entrance that is filled with light softened dentin. Probing of the cavity floor and percussion of tooth 47 are painless. Probing of the cavity walls is painful. What diagnosis can be made in this case?
Question 51Answer

a.
Chronic median caries

b.
Acute median caries

c.
Acute deep caries

d.
Chronic deep caries

e.
Chronic superficial caries

A

b.
Acute median caries

How well did you know this?
1
Not at all
2
3
4
5
Perfectly
52
Q

A child is 8 years old. There are complaints of overcrowded upper incisors. Objectively: the first molars closure is of Angle’s I class, frontal overbite is orthognathic. The 12 and 22 teeth erupt palatinally with space deficiency of 2/3 of the tooth crown. The 11 and 21 teeth are 10 mm each in cross-section. The child has inherited father’s facial type with prognathism and macrodontia of the central incisors. Choose the preventive treatment, considering this hereditary pathology:
Question 52Select one:

A.
Hotz serial extraction to reduce the dental arch

B.
Jaw expansion to provide the space for the 12 and 21 teeth

C.
Extraction of the 12 and 21 teeth to reduce the dental arch

D.
Massage of the 12 and 21 teeth area to stimulate their eruption

E.
Filing down of the 11 and 21 approximal surfaces to provide the space for the 12 and 22 teeth

A

A.
Hotz serial extraction to reduce the dental arch

Hotz serial extraction involves the strategic removal of deciduous teeth to guide the eruption of permanent teeth, helping to manage crowding and align the dental arch. This approach is particularly beneficial when there’s a potential for space deficiency and inherited traits affecting tooth size and alignment.

How well did you know this?
1
Not at all
2
3
4
5
Perfectly
53
Q

A 32-year-old patient addressed a dentist with complaints of inability to close his mouth. Objectively the mouth is half-open, the chin is protruding forwards and is displaced to the left. Such condition occurred after the mouth was opened wide. What is the most likely diagnosis?
Question 53Select one:

A.
Anterior right-sided mandibular dislocation

B.
Anterior left-sided mandibular dislocation

C.
Anterior bilateral mandibular dislocation

D.
Posterior left-sided mandibular dislocation

E.
Posterior right-sided mandibular dislocation

A

A.
Anterior right-sided mandibular dislocation

How well did you know this?
1
Not at all
2
3
4
5
Perfectly
54
Q

During a visit to a dental orthopedist, a patient underwent fitting of the metal frame for the porcelain-fused-to-metal dental bridge. What is the next clinical stage of denture-making in this case?
Question 54Answer

a.
Glazing

b.
Fixation of the porcelain-fused-to-metal dental bridge

c.
Determining the central ratio of the jaws

d.
Obtaining working and auxiliary casts

e.
Checking the design of the porcelain-fused-to-metal dental bridge

A

hecking the design of the porcelain-fused-to-metal dental bridge

How well did you know this?
1
Not at all
2
3
4
5
Perfectly
55
Q

An 18-year-old girl came to a dentist to check the quality of fissure sealing that had been performed one year ago. Objectively: the sealant is completely retained in the 17, 16, 26, 27, 37, and 47. No sealant was detected in the 36 and 46, in the distal longitudinal fissures of these teeth there is softened enamel. Fedorov-Volodkina Hygiene Index is 2,5. What tactics regarding the 36 and 46 should the dentist choose in this case?
Question 55Select one:

A.
Preventive filling

B.
Electrophoresis of calcium-containing solution

C.
Repeated noninvasive sealing

D.
Applications with fluorine-containing gel

E.
Invasive sealing

A

A.
Preventive filling

Performing a preventive filling is indicated when there is evidence of softened enamel, indicating the need for intervention to prevent further decay and maintain the integrity of the tooth.

How well did you know this?
1
Not at all
2
3
4
5
Perfectly
56
Q

A 63-year-old man complains of pain in the area of maxillary mucogingival fold caused by using a removable laminar denture. Objectively, in the area of the mucogingival fold there is a trophic ulcer with swollen margins and hemorrhaging floor. Make the diagnosis:
Question 56Select one:

A.
Denture-related stomatitis

B.
Toxic infectious stomatitis

C.
Toxic chemical stomatitis

D.
Greenhouse effect

E.
Allergic contact stomatitis

A

A.
Denture-related stomatitis

How well did you know this?
1
Not at all
2
3
4
5
Perfectly
57
Q

During application of infraorbital anaesthesia the patient developed a postinjection hematoma. What vessel had been damaged?
Question 57Select one:

A.
Maxillary artery

B.
Infraorbital artery

C.
Temporal artery

D.
Pterygoid venous plexus

E.
Palatine artery

A

B.
Infraorbital artery

How well did you know this?
1
Not at all
2
3
4
5
Perfectly
58
Q

A 42-year-old patient has been hospitalized with Le Fort II fracture of maxilla. Select an appliance for the treatment of this patient:
Question 58Select one:

A.
Penn-Brown

B.
Rudko

C.
Yadrova

D.
Kulagin

E.
Zbarzh

A

E.
Zbarzh

How well did you know this?
1
Not at all
2
3
4
5
Perfectly
59
Q

A 25-year-old woman made an appointment with the dental surgeon for oral cavity sanation. Objectively the crown of tooth 37 is destroyed by 2/3. Gingival mucosa around tooth 37 is without changes. What anesthesia should the dental surgeon give to the patient for the procedure of tooth extraction?
Question 59Select one:

A.
Mental nerve block

B.
Mandibular anesthesia

C.
Mandibular and buccal anesthesia

D.
Tuberal anesthesia

E.
Intraoral infraorbital nerve block

A

C.
Mandibular and buccal anesthesia

How well did you know this?
1
Not at all
2
3
4
5
Perfectly
60
Q

A man was diagnosed with hard palate abscess. What approach should be chosen for abscess dissection?
Question 60Select one:

A.
Pus aspiration with a syringe

B.
Triangular dissection of the hard palate area

C.
Linear dissection parallel to the hard palate raphe

D.
Abscess puncture

E.
Linear dissection perpendicular to the hard palate raphe

A

B.
Triangular dissection of the hard palate area

How well did you know this?
1
Not at all
2
3
4
5
Perfectly
61
Q

A 7-year-old practically healthy child was undergoing the carious cavity preparation of tooth 46 due to acute median caries. During this procedure the mesio-buccal pulp horn was accidentally exposed. What treatment would be optimal in this case?
Question 61Select one:

A.
Vital amputation

B.
Vital extirpation

C.
Biological approach

D.
Devital amputation

E.
Devital extirpation

A

C.
Biological approach

biological approach involves techniques that aim to preserve pulp vitality, and it is often preferred in pediatric cases.

62
Q

A 50-year-old patient has a defect of the lower dental arch. It is planned to make an implant-supported bridge for its restoration. X-ray picture shows that the height of the bone mass from projection of mandibular canal up to the top of alveolar crest is 2 cm. What type of implant should be applied?
Question 62Select one:

A.
Subperiosteal

B.
Conical

C.
Plate-form

D.
Endodontic-endoosseous

E.
Threaded

A

E.
Threaded

63
Q

A 24-year-old man complains of painful and bleeding gums. The condition onset was 3 days ago after the patient had a case of acute respiratory disease. Objectively the gingival mucosa is swollen, bright-red, bleeds on probing, painful on palpation; tips of the gingival papilla are rounded; soft dental deposit is observed. X-ray shows no changes in the bone. What is the most likely diagnosis?
Question 63Select one:

A.
Ulcerative gingivitis

B.
Periodontitis

C.
Atrophic gingivitis

D.
Catarrhal gingivitis

E.
Hypertrophic gingivitis

A

D.
Catarrhal gingivitis

64
Q

A 28-year-old woman came to the dentist for oral cavity sanation. She was diagnosed with acute deep caries of tooth 24. She has a history of seasonal allergic rhinitis and drug allergy that manifests as a skin rash and Quincke edema. She does not remember whether she has been given local anesthesia at the dentist’s office previously. What tactics should the dentist choose regarding the anesthesia in this case?
Question 64Select one:

A.
Perform an anesthetic scratch test in the dental chair immediately before giving anesthesia

B.
Use ether anesthetic

C.
Perform a cutaneous anesthetic allergy test in the dental chair immediately before giving anesthesia

D.
Referral to the allergologist for consultation and anesthetic allergy testing

E.
Use amide local anesthetic

A

D.
Referral to the allergologist for consultation and anesthetic allergy testing

65
Q

32-year-old woman complains of tumor-like growth in the mucosa of her left cheek. Locally: buccal mucosa is of normal color. In the distal area there is a rounded elongated growth, soft and elastic, attached to a pedicle sized 0,5х1,5 cm. Make the provisional diagnosis:
Question 65Select one:

A.
Fibroma

B.
Lipoma

C.
Hemangioma

D.
Papilloma

E.
Pleomorphic adenoma

A

D.
Papilloma

66
Q

A 6-year-old child presents with weakness, headache, painful swallowing, and a fever of 38–39 °C that are observed for the third day already. Examination detects several round erosions up to 2 mm in diameter with a bright red bottom on the hyperemic mucosa of the palatine arches, tonsils, and partially on the soft palate. Submandibular lymph nodes are enlarged and mildly painful to palpation. What is the most likely diagnosis in this case?
Question 66Answer

a.
Diphtheria

b.
Acute herpetic stomatitis

c.
Erythema multiforme exudativum

d.
Infectious mononucleosis

e.
Herpetic tonsillitis

A

e.
Herpetic tonsillitis

67
Q

A 60-year-old patient came to the maxillofacial inpatient department. He complains of pain, bleeding, and chewing problems. He has a history of mandibular trauma at the level of the central incisors. Objectively the traumatized place is swollen, mouth opening is unrestricted. He was diagnosed with medial mandibular fracture. Both jaws are edentulous. What splint would be optimal in this case?
Question 67Select one:

A.
Gunning gingival splint

B.
Port gingival splint

C.
Limberg gingival splint

D.
Rudko appliance

E.
Vankevych dentogingival splint

A

B.
Port gingival splint

68
Q

A 27-year-old man received a trauma of the lower jaw. After X-ray examination he was diagnosed with a displaced unilateral open mandibular fracture at the level of the mental foramen. The patient has retained all his teeth. The fracture line passes between teeth 34 and 35. What splint should be used in this case?
Question 68Select one:

A.
Splint with an inclined plane

B.
Two-jaw splint with a spreader arch

C.
Splint with a corrective loop

D.
Two-jaw splint with wire loops

E.
Smooth dental brace

A

D.
Two-jaw splint with wire loops

69
Q

A 2.5-year-old child has fever up to 38.5°C, low appetite, rashes in the oral cavity. The disease onset was 3 days ago. Objectively: the skin of the perioral area is covered in scarce vesicles with clear content. Within the oral cavity on the buccal and lingual mucosa there are sharply painful erosions, 2-3 mm in size, with white coating and hyperemic crown. The gums are swollen, hyperemic. The submandibular lymph nodes are enlarged, painful on palpation. Make the diagnosis:
Question 69Select one:

A.
Erythema multiforme exudativum

B.
Stomatitis with the background of infectious mononucleosis

C.
Stomatitis with the background of chickenpox

D.
Acute herpetic stomatitis

E.
Stevens-Johnson syndrome

A

D.
Acute herpetic stomatitis

70
Q

A 48-year-old man came to a dental surgeon for extraction of the roots of tooth 37. What instrument must be used for this procedure?
Question 70Answer

a.
S-shaped forceps

b.
Beak-shaped non-crushing forceps

c.
Bayonet forceps

d.
Beak-shaped crushing forceps

e.
Beak-shaped forceps, curved in a plane

A

d.
Beak-shaped crushing forceps

71
Q

34-year-old man complains of acute spasmodic pain in the region of his upper jaw on the left that is aggravating when affected by cold stimuli. Toothache irradiates to the ear and temple. He had acute toothache in the 27 tooth one year ago, but he did not consult a dentist. Pain recurred three days ago. Objectively: the 27 tooth has a carious cavity communicating with the dental cavity. Probing of the opened carious cavity is extremely painful. X-ray picture shows widening of periodontal fissure at the root apex of the 27 tooth. What is the most likely diagnosis?
Question 71Select one:

A.
Acute diffuse pulpitis

B.
Exacerbation of chronic gangrenous pulpitis

C.
Exacerbation of chronic fibrous periodontitis

D.
Exacerbation of chronic granulating periodontitis

E.
Exacerbation of chronic fibrous pulpitis

A

E.
Exacerbation of chronic fibrous pulpitis

72
Q

A 5.5-year-old child is undergoing preventive examination. There are no complaints from the patient. Objectively: in the precervical area of buccal surface of the 75 tooth there is an enamel patch with loss of natural glossiness. The enamel surface is coarse, painless on probing. Tooth percussion is painless. Thermodiagnosis is negative. Damaged enamel stains with 2% water solution of methylene blue. Make the diagnosis:
Question 72Select one:

A.
Acute superficial caries

B.
Chronic initial caries

C.
Fluorosis

D.
Acute initial caries

E.
Enamel hypoplasia

A

D.
Acute initial caries

73
Q

A 42-year-old man came to the dental surgeon for oral sanation. After application of anesthesia, the patient suddenly developed nausea, acute weakness, constricting sensation behind the sternum, and cardiac pain. The patient’s skin is pale and covered in cold clammy sweat. The patient started vomiting, lost his consciousness, and developed seizures. The patient’s pupils are unresponsive to the light. The pulse is thready, blood pressure cannot be measured. Make the diagnosis:
Question 73Select one:

A.
Anaphylactic shock

B.
Epileptic seizure

C.
Traumatic shock

D.
Unconsciousness

E.
Collapse

A

A.
Anaphylactic shock

74
Q

A 57-year-old patient adressed a prosthodontic clinic to have a denture made for him. After objective examination the patient was prescribed single-unit crowns for the 46, 47, 36, 37 teeth. What maximal angle of a tooth stump can be allowed during preparation?
Question 74Select one:

A.
3-5$^o$

B.
10-12$^o$

C.
Stump walls should be parallel to each other

D.
7-8$^o$

E.
5-6$^o$

A

A.
3-5$^o$

75
Q

A 65-year-old woman complains of a painless neoplasm on her upper lip that slowly enlarges. Objectively on the upper lip there is a neoplasm 1x1 cm in size that looks like a gray-red nodule with a funnel-shaped indent in its center, filled with a dense keratinized mass. Make the provisional diagnosis:
Question 75Select one:

A.
Flat leukoplakia

B.
Verrucous precancer

C.
Bowen’s disease

D.
Keratoacanthoma

E.
Cutaneous horn

A

D.
Keratoacanthoma

76
Q

A 50-year-old man came to a dentist to have dentures made for him. During preparation of teeth, the patient complained of a headache and dizziness, he became irritable and anxious and suddenly fainted. Objectively, the pupils were dilated and unresponsive to light, tonic convulsions appeared and became clonic, salivation increased. In a few minutes, the convulsions stopped. The patient experiences mental confusion. What emergency condition most likely occurred in this case?
Question 76Answer

a.
Hypoglycemic coma

b.
Anaphylactic shock

c.
Epilepsy attack

d.
Hyperglycemic coma

e.
Unconsciousness

A

c.
Epilepsy attack

77
Q

A 9-year-old girl complains of peeling lips and sensation of dryness, strain and burning in them. Objectively: on the red border from angle to angle of the mouth and from Klein area (wet-dry line) to the red border middle there are gray scales. The scales can be removed without erosions. Make the preliminary diagnosis:
Question 77Select one:

A.
Allergic contact cheilitis

B.
Atopic cheilitis

C.
Meteorological cheilitis

D.
Exfoliative cheilitis, exudative form

Eform.
Exfoliative cheilitis, dry form

A

E.
exfoliatingexfoliatife .dry form

78
Q

37-year-old patient complains of bleeding gums, bad breath, loose teeth, difficulties with chewing food. Objectively: gums are bluish-red in colour; periodontal pockets are 6 mm deep and contain serous substance; class II teeth mobility; moderate deposits of subgingival and supragingival dental calculus; Oral Hygiene Index equals 3 points; there is traumatical occlusuion along all the length of the dental arches. What is the provisional diagnosis?
Question 78Select one:

A.
Generalized periodontitis, stage II, chronic

B.
Generalized periodontitis, stage II, acute

C.
Histiocytosis X

D.
Hypertrophic gingivitis

E.
Generalized periodontitis, stage I, acuteacute

A

A.
Generalized periodontitis, stage II, chronicchronic

79
Q

A 50-year-old woman complains of pain and creaking in her temporomandibular joint. Objectively there are multiple premature occlusal contacts and localized pathologic tooth wear. The teeth are stable, with exposed cervices; the gingival margin is hyperemic. Selective teeth shaving is planned. How many visits to the dentist are necessary for the full procedure?
Question 79Select one:

A.
2-3 visits at one-day intervals

B.
2-3 visits at one-month intervals

C.
A single visit

D.
3-4 visits at one-week intervals

E.
1-2 visits within a yearyear

A

3-4 visits at one-week intervalsintervals

80
Q

A 45-year-old patient has a bilateral mental mandibular fracture with a typical displacement of fragments. What actions should be taken to eliminate asphyxia?Question 80Select oneone
:A.Tracheal intubation through the nosemouthmouthmouthmouth
B.Tracheostomy
C.Pull out and pierce the tongue
D.Artificial pulmonary ventilationE.Tracheal intubation through the mouthmouthmouthmouth

A

C.
Pull out and pierce the tonguetongue

81
Q

A 13-year-old boy presents with enlarged lymph nodes on the anterior surface of his neck and in the parotid and submandibular regions on the right. The lymph nodes are dense, painless, mobile, 2–2.5 cm in diameter. Hypertrophy of palatine tonsils is detected. The oral cavity is sanated. Sternberg giant cells were detected in the punctate. Make the clinical diagnosis.
Question 81Answer

a.
Lymphogranulomatosis

b.
Chronic hyperplastic lymphadenitis

c.
Burkitt lymphoma

d.
Infectious mononucleosis

e.
Acute leukemialeukemialeukemialeukemialeukemialeukemialeukemialeukemia

A

a.
Lymphogranulomatosis

82
Q

A woman came to a dentist with complaints of periodic pain in the area of her tooth 48. Extraction of tooth 48 is indicated. What forceps must be used for the tooth extraction in this case?
Question 82Answer

a.
S-shaped right-sided forceps

b.
Beak-shaped forceps, bent along the plane

c.
Beak-shaped forceps, bent along the rib

d.
Bayonet-like forceps

e.
Beak-shaped forceps with converging beaksbeaksbeaksbeaks

A

Beak-shaped forceps, bent along the planeplane

83
Q

Dental splint is being designed in a prosthodontics clinic for a 39-year-old patient with generalized periodontitis, II degree. Gnathodynamometer is used to measure the periodontium load resistance. What anatomico-functional data are obtained with this method?
Question 83Select one:

A.
Masticatory force

B.
Periodontium pliancy

C.
Masticatory muscles tone

D.
Masticatory efficiency

E.
Masticatory pressure

A

E.
Masticatory pressure

84
Q

One of the stages of making a removable full denture includes fitting of an impression tray and obtaining the functional impression. What material is applied to the edges of the impression tray?
Question 84Select one:

A.
Kromopan

B.
Thiodent

C.
Orthocor

D.
Plaster

E.
Stomalgin

A

C.
Orthocor

85
Q

The parents of an 8-year-old boy complain that their child suffers from acute pain in his left lower jaw, general weakness, fever of 38.6°C, and problems with mouth opening and swallowing. Objectively, the face is asymmetrical because of edema in the submandibular region and in the lower third of the left cheek. Mucosa in the area of teeth 73, 74, and 75 is edematous on the both sides of the alveolar process. The teeth are mobile, their percussion is painful, the Vincent’s sign is observed. What is the most likely diagnosis in this case?
Question 85Answer

a.
Acute odontogenic osteomyelitis

b.
Odontogenic phlegmon of the submandibular region

c.
Non-odontogenic acute purulent lymphadenitis

d.
Exacerbation of chronic periodontitis

e.
Inflammatory infiltrate of the left cheek

A

a.
Acute odontogenic osteomyelitis

86
Q

A patient needs endodontic treatment of tooth 21. The canal is being prepared with manually operated endodontic instruments. Name one such instrument made by means of conical spiral threading (turning) of a steel wire with a round cross-section (milling).
Question 86Answer

a.
K-reamer

b.
K-file

c.
Spreader

d.
H-file

e.
Plugger

A

d.
H-file

87
Q

What is used to transfer to the articulator the data about the position of the upper jaw in relation to the hinge axis of the temporomandibular joint?
Question 87Answer

a.
Auxiliary impressions

b.
Facebow

c.
Occluder

d.
Parallelometer

e.
Bite blocks

A

b.
Facebow

88
Q

During the treatment of ulcerative gingivitis, a 38-year-old man was locally prescribed a drug that belongs to the hydrolase enzymes and is a thermostable protein contained in tears, saliva, and breast milk as an innate resistance factor of human body. Name this drug.
Question 88Answer

a.
Lysozyme

b.
Interleukin

c.
Interferon

d.
Imaninum

e.
Penicillin

A

a.
Lysozyme

89
Q

A 21-year-old man came to a dentist with complaints of difficulty opening his mouth and a change in the appearance of his face observed over the past 2 years. He has a history of a facial injury sustained several years ago. Objectively, the right half of the face is flattened, the chin is displaced to the left. Palpation detects decreased excursion of the articular head of the mandible on the left. Mouth opening is limited to 1 cm. Make the provisional diagnosis.
Question 89Answer

a.
Anterior right-sided dislocation of the mandible

b.
Ankylosis of the left temporomandibular joint

c.
Arthrosis of the left temporomandibular joint

d.
Acute arthritis of the left temporomandibular joint

e.
Exacerbation of chronic arthritis of the temporomandibular joint on the left

A

b.
Ankylosis of the left temporomandibular joint

90
Q

A 50-year-old man came to a dentist to have dentures made for him. During preparation of teeth, the patient complained of a headache and dizziness, he became irritable and anxious and suddenly fainted. Objectively, the pupils were dilated and unresponsive to light, tonic convulsions appeared and became clonic, salivation increased. In a few minutes, the convulsions stopped. The patient experiences mental confusion. What emergency condition most likely occurred in this case?
Question 90Answer

a.
Hypoglycemic coma

b.
Hyperglycemic coma

c.
Unconsciousness

d.
Epilepsy attack

e.
Anaphylactic shock

A

d.
Epilepsy attack

91
Q

A 28-year-old woman came to a dentist for oral cavity sanation. She was diagnosed with acute deep caries of tooth 24. The patient has a history of seasonal allergic rhinitis and drug allergy that manifests as a skin rash and Quincke’s edema. The patient does not remember whether she was previously receiving local anesthesia during dental procedures. What should be the dentist’s tactics regarding anesthesia in this case?
Question 91Answer

a.
Administer anesthesia using an amide anesthetic

b.
Refer to an allergist for consultation and anesthetic allergy testing

c.
Conduct a scarification test for the anesthetic in the dental chair immediately before administering anesthesia

d.
Administer anesthesia using an ether anesthetic

e.
Conduct an intradermal test for the anesthetic in the dental chair immediately before administering anesthesia

A

b.
Refer to an allergist for consultation and anesthetic allergy testing

92
Q

Preventive dental examination of a 5-year-old boy shows that the cusps of the temporary canines on the lower jaw have not worn off, which causes the lower jaw to shift forwards. What measures are necessary to prevent the development of a pathological bite in this child?
Question 92Answer

a.
Making a maxillary plate with a vestibular arch

b.
Dynamic observation with visits to the dentist every 6 months

c.
Making a mandibular plate with occlusal overlays and a screw

d.
Miogymnastic exercises

e.
Filing down the cusps of temporary canines

A

e.
Filing down the cusps of temporary canines

93
Q

ays after the extraction of tooth 46, a 29-year-old man developed complaints of a dull pain in the area of the dental alveolus of extracted tooth 46. He presents with hyperemic and edematous mucosa in the area of the dental alveolus; no blood clot; food debris are detected. Make the provisional diagnosis:
Question 93Select one:

A.
Acute suppurative alveolitis

B.
Neuritis of the alveolar nerve

C.
Chronic local osteomyelitis

D.
Acute serous alveolitis

E.
Acute necropurulent alveolitis

A

D.
Acute serous alveolitis

94
Q

A 32-year-old man came to a dentist with complaints of skin redness and a swelling in the area of his left cheek. Two months ago, he noticed a dense spot on his left cheek that increased in size over time. The skin over the spot has become bluish-red and a softening has formed in its center. Objectively, a pathological focus 2.3x1.4 cm in size with clear borders is observed on the skin of the left cheek. The skin there is sharply thinned-out, bluish, shiny, and forms folds and irregularities. Intraorally, a palpable cord is detected that leads from tooth 46 to the lesion on the cheek. Tooth 46 is filled. Make the provisional diagnosis.
Question 94Answer

a.
Carbuncle

b.
Phlegmon of the buccal region

c.
Chronic lymphadenitis

d.
Migratory subcutaneous granuloma

e.
Furuncle

A

d.
Migratory subcutaneous granuloma

95
Q

A 28-year-old man came to the maxillofacial surgery department. He complains of a cutaneous neoplasm in his right submandibular region. The neoplasm appeared 2 years ago. In the process of shaving, the neoplasm is frequently injured. The following provisional diagnosis has been made: papilloma. In the neoplastic area, examination shows an ulcer against the background of hyperemic skin. What tactics should the dental surgeon choose?
Question 95Answer

a.
Anti-inflammatory therapy followed by observation

b.
Prevention of malignant transformation of the papilloma followed by removal of the papilloma

c.
Anti-inflammatory therapy followed by removal of the papilloma

d.
Removal of the papilloma

e.
Removal of the papilloma and postoperative radiation therapy

A

b.
Prevention of malignant transformation of the papilloma followed by removal of the papilloma

96
Q

A 79-year-old female patient consulted a prosthodontist about denture replacement. The patient has a history of a stroke. Objectively: acute irregular atrophy of the alveolar processes of both jaws is present; mucous membrane of the oral cavity is dry and nonmobile. The previous dentures cannot be fixed. What is the most appropriate prosthetic construction?
Question 96Select one:

A.
Implant-supported dentures

B.
Dentures with shortened borders

C.
Dentures with elastic lining

D.
Dentures with metal bases

E.
Dentures with extended borders

A

C.
Dentures with elastic lining

97
Q

A 30-year-old woman complains of a painless swelling in the area of her right lower jaw. The swelling appeared 6 months ago and is slowly increasing in size. Objectively, on the body of the lower jaw on the right in the area of teeth 45 and 46, there is a dense neoplasm 2.5x1.5 cm in size, painless to palpation. The skin and mucosa over the neoplasm are mobile and without discoloration. The oral cavity is sanated. The teeth on the lower jaw are intact. The sensitivity of the lower lip is unchanged. Regional lymph nodes are not palpable. X-ray detects a polycystic formation with clear boundaries in the area of the body of the mandible on the right. Make the provisional diagnosis.
Question 97Answer

a.
Osteosarcoma

b.
Odontoma

c.
Odontogenic fibroma

d.
Follicular cyst

e.
Ameloblastoma

A

e.
Ameloblastoma

98
Q

A 12-year-old girl complains of periodic bleeding in the area of her lower right tooth. Objectively, the masticatory surface of tooth 46 has a carious cavity filled with a soft red tissue. Superficial probing is mildly painful, while deeper probing causes pain and bleeding. The tooth is almost unresponsive to thermal stimuli. Percussion of tooth 46 is painless. X-ray detects no pathological changes in the periodontal region of the tooth. What diagnosis can be made in this case?
Question 98Answer

a.
Chronic fibrous pulpitis

b.
Chronic localized hypertrophic gingivitis

c.
Chronic hypertrophic pulpitis

d.
Chronic gangrenous pulpitis

e.
Chronic granulating periodontitis

A

c.
Chronic hypertrophic pulpitis

99
Q

A patient is to undergo endodontic treatment of tooth 21 due to exacerbation of chronic pulpitis. The dentist chooses tools for the treatment. What is the function of a spreader?
Question 99Answer

a.
Widening of root canals

b.
Pulp removal from root canals

c.
Filling of root canals

d.
Assessment of root canal patency

e.
Root canal length measuring

A

c.
Filling of root canals

100
Q

A hit to the face has forced the upper central incisors of a 15-year-old boy to sink into the jaw to the half of their crown height. What treatment tactics should the dentist choose?
Question 111Select one:

A.
Transplantation of 11 and 21 into the dentition, endodontic treatment

B.
Dynamic observation, endodontic treatment of 11 and 21 if necessary

C.
Extraction of 11 and 21, their replacement with dentures

D.
Replantation of 11 and 21 into the dentition, ligation

E.
Surgical repositioning of 11 and 21, teeth immobilization, endodontic treatment if necessary

A

E.
Surgical repositioning of 11 and 21, teeth immobilization, endodontic treatment if necessary

101
Q

After extraction of tooth 27 the dental surgeon suspected that the maxillary sinus was perforated. To clarify this provisional diagnosis the doctor desided to conduct an oronasal communication test (Valsalva maneuver). What manipulations are necessary for this test?
Question 100Select one:

A.
Ask the patient to blow through the mouth, while observing the socket of 27

B.
Pinch the nostrils together to occlude the patient’s nose and ask the patient to blow through the mouth, while observing the socket of 27

C.
Ask the patient to blow through the nose, while observing the socket of 27

D.
Pinch the nostrils together to occlude the patient’s nose and ask the patient to blow through the nose, while observing the socket of 27

E.
Ask the patient to inhale through the nose, pinch the nostrils together, and exhale through the mouth

A

D.
Pinch the nostrils together to occlude the patient’s nose and ask the patient to blow through the nose, while observing the socket of 27

102
Q

A 39-year-old woman complains of general malaise, headache, fever of 38°C, and pain in the oral cavity. Similar condition occurs periodically, more often in autumn and spring. Objectively, her sharply hyperemic and swollen mucosa of the cheeks, lips, and soft palate has erythematous spots, burst blisters, and erosions covered with fibrinous plaque. Nikolsky’s sign is negative. Hypersalivation is observed. Submandibular lymph nodes are enlarged, mobile, and somewhat painful to palpation. What is the most likely diagnosis in this case?
Question 101Answer

a.
Erythema multiforme exudativum

b.
Lichen ruber planus, erosive-ulcerative form

c.
Pemphigus vulgaris

d.
Chronic recurrent aphthous stomatitis

e.
Chronic recurrent herpes

A

a.
Erythema multiforme exudativumErythema multiforme exudativum

103
Q

The parents of an 11-year-old boy came to a dentist with complaints that their child’s gums are painful and bleeding during eating, the child has bad breath and a fever of 37.4°С. The boy has a recent history of an acute respiratory viral infection. Objectively, the gingival mucosa is brightly hyperemic and bleeds easily when touched. The apices of the interdental papillae are covered with a grayish necrotic plaque. The removal of the plaque exposes an ulcerated surface. There is a large amount of dental plaque. What is the causative agent of this disease?
Question 102Answer

a.
Streptococci

b.
Staphylococci

c.
Anaerobic microflora

d.
Herpes virus

e.
Yeast-like fungi

A

c.
Anaerobic microflora

104
Q

A patient with ischemic heart disease has developed ventricular fibrillation. What is the first-priority therapeutic action?
Question 103Select one:

A.
Adrenaline injection

B.
Lidocaine injection

C.
Potassium chloride injection

D.
Electric defibrillation

E.
Novocaine amide injection

A

D.
Electric defibrillation

104
Q

A 28-year-old woman is diagnosed with chronic generalized periodontitis, II degree. The doctor prescribed her a mouthwash with chlorhexidine gluconate as a part of complex therapy. This drug belongs to the following group of antiseptics:
Question 104Select one:

A.
Halogens

B.
Dyes

C.
Acids and alkalis

D.
Detergents

E.
Oxidants

A

D.
Detergents

104
Q

A 24-year-old woman came to a dentist to receive sanation. Objectively on the masticatory surface of the 37 tooth there is a deep carious cavity connected with the dental cavity. The cavity probing is painless, no reaction to thermal stimuli is observed in the tooth, percussion is painless. EOD is 108 microamperes. X-ray shows traces of filling material in the rooth canal of the 37 tooth, periodontal fissure is enlarged and deformed. Make the diagnosis:
Question 105Select one:

A.
Chronic granulating periodontitis of the 37 tooth

B.
Exacerbation of chronic granulomatous periodontitis of the 37 tooth

C.
Chronic fibrous pulpitis of the 37 tooth

D.
Chronic fibrous periodontitis of the 37 tooth

E.
Chronic granulomatous periodontitis of the 37 tooth

A

D.
Chronic fibrous periodonti

105
Q

33-year-old man, a metalworker, complains of pain and itching in the gums, gingival hemorrhages intensifying during tooth brushing. The onset of the disease was 1 year ago. Objectively: the gums in the area of upper and lower frontal teeth are hyperemic, swollen, and cyanotic. There are significant mineralized deposits on the teeth; the periodontal sockets are 3 mm deep and produce small amount of serous discharge. What is the most likely diagnosis?
Question 106Select one:

A.
Exacerbation of chronic generalized periodontitis, II class

B.
Chronic generalized periodontitis, I class

C.
Generalized periodontosis, I class

D.
Chronic localized periodontitis, I class

E.
Acute localized periodontitis, II class

A

B.
Chronic generalized periodontitis, I class

105
Q

A 27-year-old woman complains of destruction of her upper front teeth. She was diagnosed with chronic deep caries of teeth 11 and 21, Black’s class IV. Aesthetic restoration of teeth 11 and 21 is planned. What material has optimal combination of strength and aesthetical properties?
Question 107Answer

a.
Hybrid composites

b.
Liquid composites

c.
Macrofilled composites

d.
Microfilled composites

e.
Compomers

A

a.
Hybrid composites

106
Q

arents of a 2.5-year-old child complain of gradual destruction of the upper front teeth of their child for the last several months. Objectively there are carious cavities within mantle dentin on the contact and vestibular surfaces of 52, 51, 61, and 62. The cavities are filled with softened pigmented dentin that can be easily removed with dental excavator. Make the provisional diagnosis:
Question 108Select one:

A.
Acute median caries

B.
Acute deep caries

C.
Chronic superficial caries

D.
Chronic median caries

E.
Chronic deep caries

A

A.
Acute median caries

107
Q

A 40-year-old woman complains of reddened lower lip, as well as pain and heartburn during eating that have been observed for the last several months. She has chronic polyarthritis and spends a lot of time in the sunlight. Objectively, a symmetrical, markedly red patch with a multi-contour outline is detected on the vermilion border of the lower lip. The lesion is moderately infiltrated. There are white-gray scales tightly attached to its surface. The removal of the scales provokes pain and bleeding. At the periphery of the lesion focus, there are strips of cloudy gray epithelium. Make the provisional diagnosis.
Question 109Answer

a.
Lichen ruber planus

b.
Actinic cheilitis

c.
Lupus erythematosus

d.
Exfoliative cheilitis

e.
Leukoplakia

A

c.
Lupus erythematosus

108
Q

A man with hemophilia came to a dentist with complaints of pain in his lower left tooth. A root fracture was diagnosed in tooth 36. Tooth extraction is planned. What will be the dentist’s tactics in this case?
Question 110Answer

a.
Perform the tooth extraction in the maxillofacial surgery department

b.
Perform the tooth extraction in a dental polyclinic with pre-operative care

c.
Perform the tooth extraction in a dental polyclinic with pre- and post-operative care

d.
Refer to the hematology department for the tooth extraction

e.
Perform the tooth extraction in the vascular surgery department

A

**
d.
Refer to the hematology department for the tooth extraction
**

108
Q

During carious cavity preparation in a 20-year-old man, the pulp-chamber floor was accidentally perforated and horn of the pulp was exposed. On the carious cavity floor there is a point-like puncture surrounded with white predentin. Pink pulp can be seen through the perforation, pulp probing is acutely painful. What treatment should be given to the patient?
Question 112Select one:

A.
Devital extirpation

B.
Vital extirpation

C.
Devital amputation

D.
Biological approach

E.
Vital amputation

A

D.
Biological approach

108
Q

The parents of a 10-year-old boy came to a dentist with complaints of a swelling on the right side of the child’s neck. The swelling appeared 4 days ago. During examination, a round, dense, mildly painful formation is palpable in the right upper neck region. The formation is not fused with the adjacent tissues, 3 cm in diameter, without skin discoloration above it. Intraorally, destroyed tooth 85 was detected. Make the provisional diagnosis.
Question 113Answer

a.
Acute serous non-odontogenic lymphadenitis

b.
Acute serous odontogenic lymphadenitis

c.
Lateral cyst of the neck

d.
Atheroma of the right upper neck region

e.
Acute purulent odontogenic lymphadenitis

A

b.
Acute serous odontogenic lymphadenitis

109
Q

emovable complete laminar denture is being made for a 63-year-old edentulous man. At the stage of placing the artificial teeth in the dental articulator it is necessary to determine sagittal articular angle. This angle equals:
Question 114Select one:

A.
$15$-$20^о$

B.
$20$-$40^о$

C.
$40$-$50^о$

D.
$5$-$15^о$

E.
$20$-$25^о$

A

B.
$20$-$40^о$

110
Q

A 22-year-old patient complains of a painful swelling in the right parotid gland. A week earlier the patient received a cheek abrasion that healed under the purulent crust. Over the past two days the patient had been observing progressing pain and fever up to 38,6oC. Objectively: there is a soft tissue edema in the right parotid region, the skin is slightly strained, without discoloration. There is a dense painful infiltration 2,5x3,5 cm large, the skin over it exhibits limited mobility. The mouth can be fully opened, the mucous membrane around the orifice of the salivary duct is unchanged, saliva is transparent. What is the most likely diagnosis?
Question 115Select one:

A.
Epidemic parotitis

B.
Acute lymphadenitis

C.
Abscess of the parotid-masseteric region

D.
Exacerbation of chronic parotitis

E.
Acute non-epidemic parotitis

A

B.
Acute lymphadenitis

111
Q

Tooth 46 is being endodontically treated for chronic fibrous periodontitis. Its root canals are narrow and sclerosed. What tool should be used to widen the root canals in this case?
Question 116Answer

a.
Orthophosphoric acid

b.
Sodium hypochlorite

c.
Chlorhexidine bigluconate

d.
Ethylenediaminetetraacetic acid

e.
Aminocaproic acid

A

d.
Ethylenediaminetetraacetic acid

111
Q

A 13-year-old girl complains of a sharp pain when biting on her tooth 21. The pain developed after an injury. Objectively, the crown of tooth 21 has two fracture lines, the fragments are slightly displaced, blood is oozing from the pulp chamber, percussion is painful. X-ray shows a diagonal fracture of the crown and root of tooth 21. The depth of the subgingival edge of the fracture line is over 4 mm. What should be the dentist’s tactics regarding tooth 21?
Question 117Answer

a.
Filling

b.
Extraction

c.
Splinting

d.
Monitoring for the next month

e.
Replantation

A

b.
Extraction

112
Q

A 28-year-old man presents with profuse caseous coating on the posterior third of the back of his tongue, soft palate, tonsils, and posterior wall of the pharynx. Submandibular, submental and deep cervical lymph nodes have been enlarged for 4 months. Two weeks ago the patient developed intermittent fever and general fatigue. Select the correct sequence of HIV diagnosing:
Question 118Select one:

A.
Enzymoimmunoassay, immunoblotting (Western-Blot)

B.
Complete blood count, viral load

C.
Viral cultivation, enzymoimmunoassay

D.
Complete blood count, enzymoimmunoassay

E.
CD4 cell count, enzymoimmunoassay

A

**A.
Enzymoimmunoassay, immunoblotting (Western-Blot)
**

113
Q

A 25-year-old patient complains of pain caused by eating sweet, hot and cold food; pain ceases, when stimulation stops. Objectively: the adjoining surface of the 36th tooth has a deep carious cavity localised in the circumpulpar dentin. The dentin is softened. Probing of the carious cavity floor is painful. What is the most probable diagnosis?
Question 119Select one:

A.
Chronic median caries

B.
Chronic deep caries

C.
Chronic fibrous pulpitis

D.
Acute median caries

E.
Acute deep caries

A

.
Acute deep caries

114
Q

How often should a dentist’s office be thoroughly cleaned?
Question 120Select one:

A.
No less than once a month

B.
No less than once a week

C.
No less than twice a week

D.
No less than once a year

E.
No less than once every 2 weeks

A

B.
No less than once a week

115
Q

Objective examination of a 10-year-old child revealed slight hyperemia, infiltration, and dryness of the whole surface of the vermillion border. Architectonics of the lips is disturbed. Dryness and contracted sensation are observed in the lips, especially during cold seasons. Make the provisional diagnosis:
Question 121Select one:

A.
Meteorological cheilitis

B.
Cheilitis of microbial origin

C.
Atopic cheilitis

D.
Allergic contact cheilitis

E.
Exfoliative cheilitis

A

A.
Meteorological cheilitis

116
Q

Parents of a 12-year-old child are concerned about the child having white spots on the frontal teeth of the upper jaw; the spots appeared half a year ago. Objectively: there are chalky spots detected in the cervical zone vestibular surfaces of the 13, 12, 11, 21, 22, and 23 teeth. The enamel in those spots is dull; probing revealed it to be pliant and coarse. The anamnesis states short-time pain caused by chemical stimuli. What is the provisional diagnosis?
Question 122Select one:

A.
Chronic initial caries

B.
Dental fluorosis

C.
Systemic hypoplasia of enamel

D.
Acute initial caries

E.
Acute superficial caries

A

Acute initial caries

116
Q

The parents of a 7-year-old girl complain of an aesthetic defect of the child’s face. Objectively, her face is symmetrical and proportional, the profile is concave, the chin protrudes, the lower lip overlaps the upper lip, the lower incisors overlap the upper incisors, the permanent molar closure is Angle’s class 3. What bite was detected in the child?
Question 123Answer

a.
Crossbite

b.
Open

c.
Deep

d.
Mesial

e.
Distal

A

d.
Mesial

116
Q

A 64-year-old man complains of pain and mobility of his front lower teeth. Objectively: the 43, 42, 41, and 31 teeth demonstrate the III degree mobility. These teeth are planned to be extracted and immediate-insertion denture is to be made. When should this denture be put in place?
Question 124Select one:

A.
In 1-2 days after the teeth extraction

B.
In 2-4 days after the teeth extraction

C.
On the day of the teeth extraction

D.
In 3-6 days after the teeth extraction

E.
In 6-8 days after the teeth extraction

A

C.
On the day of the teeth extraction

117
Q

25-year-old patient received a trauma to the chin region. On the basis of X-ray he was diagnosed with bilateral mental fracture of mandible. Specify the direction of displacement of the minor fragment:
Question 125Select one:

A.
Upward and forward

B.
Downward and forward

C.
Downward and backward

D.
Upward and backward

E.
There is no displacement

A

C.
Downward and backward

118
Q

Removable complete dentures are being made for a 70-year-old man with completely edentulous jaws and marked progenia. What artificial teeth are not placed in such dentures?
Question 126Answer

a.
Second premolars on the upper jaw

b.
First premolars on the lower jaw

c.
Second molars on the lower jaw

d.
Second molars on the upper jaw

e.
Second premolars on the lower jaw

A

a.
Second premolars on the upper jaw

119
Q

A 45-year-old woman consults a dentist for prosthetics. Objectively: the 47, 46, 45, 35, 36, 37 teeth are missing. There is dentoalveolar vertical displacement of the 17, 16, 26, 27 teeth; the alveolar process is hypertrophied, the cervical margins are not exposed. When the teeth are closed, the distance between the occlusal surfaces of the teeth and alveolar process is approximately 6 mm. Specify the most efficient method of treatment of secondary deformation in this patient:
Question 127Select one:

A.
Hardware-associated surgical treatment

B.
Grinding

C.
Disocclusion

D.
Surgical treatment

E.
Pulp removal and teeth grinding

A

.
Pulp removal and teeth grinding

120
Q

Parents of a 3-year-old child complain that the child has a neck growth that developed 3 months after the birth. Objectively: in the upper lateral neck area there is a semicircular neoplasm with limited mobility, soft elastic consistency, no skin alterations, painless on palpation. Puncture yielded pus-like clear yellow substance. Make the provisional diagnosis:
Question 128Select one:

A.
Chronic lymphadenitis

B.
Hemangioma

C.
Specific lymphadenitis

D.
Lymphangioma

E.
Branchial cleft cyst

A

**
E.
Branchial cleft cyst**

121
Q

A 20-year-old woman complains of pain in the area of tooth 38. The pain increases during swallowing and opening her mouth wide. Objectively, the crown of tooth 38 is partially covered in mucosa, from under which a purulent exudate is being discharged. Palpation is painful. The mucosa over tooth 38 is hyperemic and swollen. Make the provisional diagnosis:
Question 129Select one:

A.
Acute catarrhal pericoronitis

B.
Acute ulcerative pericoronitis

C.
Acute serous periostitis

D.
Acute suppurative periostitis

E.
Acute suppurative pericoronitis

A

**
E.
Acute suppurative pericoronitis**

121
Q

A 7,5-year-old boy fell down on his face and damaged his front maxillary teeth. Objectively: the crowns of the 11 and 12 teeth are destroyed by 1/4. The tooth cavity is closed. Probing causes pain along the break-off line, percussion of the 11 and 12 teeth causes no pain response. Mucosa around the 11 and 12 teeth is slightly hyperemic. Radiograph of the 11 and 12 teeth shows intactness of the dental tissues and alveolar process. What tactics of a dentist would be optimal in the first hours after the injury?
Question 130Select one:

A.
Photopolymer filling

B.
Composite filling

C.
Parapulpar pin reconstruction

D.
Pulp monitoring

E.
Fabrication of an artificial crown

A

D.
Pulp monitoring

122
Q

Parents of a 4-year-old child have made an appointment with an orthodontist for preventive examination of the child. In the oral cavity: scissor bite is observed in the front area in the place of inscisor contact; distal surfaces of the second temporary molars are situated in one plane. Teeth in the dental arch are placed tightly, without intervals. Determine, what period of occlusion development is described:
Question 131Select one:

A.
Involution of temporary occlusion

B.
Late stage of developing occlusion

C.
Stability of temporary occlusion

D.
Formation of temporary occlusion

E.
Early stage of developing occlusion

A

C.
Stability of temporary occlusion

123
Q

A 52-year-old man complains of pain and clicking in the area of his left temporomandibular joint. Objectively, the face is symmetrical, palpation of the medial and lateral pterygoid muscles is painful on the left. The mouth opening is limited to 1.5 cm between the cutting edges of the central incisors. X-ray shows smooth contours of the articular surfaces, the joint space is normal. What is the most likely diagnosis in this case?
Question 132Answer

a.
Neuromuscular joint syndrome

b.
Rheumatoid arthritis

c.
Deforming arthrosis

d.
Fibrous ankylosis of the joint

e.
Acute post-traumatic arthritis

A

a.
Neuromuscular joint syndrome

124
Q

A 43-year-old man developed a sharp headache, dizziness, nausea, and visual snow during tooth extraction. The patient is excited, feels heat and trembling in the whole body. Red spots appeared on the skin of his neck and face. He presents with tachycardia and blood pressure of 190/110 mm Hg. What emergency condition has developed in the patient?
Question 133Answer

a.
Hypertensive crisis

b.
Stroke

c.
Myocardial infarction

d.
Angina pectoris attack

e.
Anaphylactic shock

A

a.
Hypertensive crisis

125
Q

After a clinical examination of a 28-year-old patient, a dentist made the following diagnosis: acute median caries of tooth 25, Black’s class I. What dental drill should be used in this case to open the carious cavity during the treatment?
Question 134Answer

a.
Wheel-shaped, diamond

b.
Spherical, diamond

c.
Cylindrical, hard-alloy

d.
Inverted cone, diamond

e.
Spherical, hard-alloy

A

b.
Spherical, diamond

125
Q

The parents of a 9-year-old girl came to a dentist complaining of an aesthetic defect in a right upper front tooth of their child. The tooth has erupted with already damaged enamel. Premature extraction of tooth 62 has been earlier performed in this child due to caries complications. Examination detects a whitish-yellow spot with clear borders on the vestibular surface of tooth 22. The enamel over the spot is smooth and glossy. Make the provisional diagnosis.
Question 135Answer

a.
Local enamel hypoplasia

b.
Fluorosis

c.
Systemic enamel hypoplasia

d.
Chronic superficial caries

e.
Acute superficial caries

A

a.
Local enamel hypoplasia

125
Q

A 34-year-old man complains of pain in the area of his right eye, headache, and body temperature rise up to 38,6°C. Two days ago the patient developed an infiltration in the lower eyelid of the right eye. Objectively the eyelids are markedly swollen, palpebral fissure is closed, conjunctiva is swollen. Exophthalmos is observed. The eyeball is immobile, vision is impaired. Make the diagnosis:
Question 136Select one:

A.
Purulent maxillary sinusitis

B.
Angular vein trombophlebitis

C.
Eyelid phlegmon

D.
Orbital phlegmon

E.
Lower eyelid abscess

A

D.
Orbital phlegmon

126
Q

[](htt

In a 3-year-old child, a dentist has detected a carious cavity on the masticatory surface of tooth 74 within the mantle dentin. The overhanging edges of the enamel are matte white. The dentin of the carious cavity is light-colored, soft, and can be removed in layers. Probing of the dentinoenamel junction is tender. Cold stimulus provokes pain that quickly stops after the stimulus is removed. What material should be used for the filling of the carious cavity in this case?
Question 137Answer

a.
Glass ionomer cement

b.
Amalgam

c.
ICON

d.
Zinc phosphate cement

e.
Composite filling material

A

a.
Glass ionomer cement

127
Q

A child is 5 years old. There are complaints of an aesthetical defect. An extraoral examination shows that the face is symmetrical, but disproportionate because of reduced lower third of the face. In the oral cavity, teeth 55, 54, 64, and 65 are lost. What dental device is necessary for this patient?
Question 138Select one:

A.
Andresen-Haupl appliance

B.
Frankel appliance

C.
Dental bridge

D.
Clasp-retained (bugel) denture

E.
Removable partial denture for the upper jaw

A

E.
Removable partial denture for the upper jaw

128
Q

The patient’s upper right central incisor needs to be extracted due to exacerbation of chronic periodontitis. What type of anesthesia should be used in this case?
Question 139Answer

a.
Tuberal and palatal

b.
Infraorbital, palatal, and incisive

c.
Tuberal and incisive

d.
Infraorbital and palatal

e.
Infraorbital and incisive

A

e.
Infraorbital and incisive

128
Q

A 50-year-old patient has median lower jaw fracture with formation of a false joint. The 38, 32, 31, 41, 42, 48 teeth are missing. The remaining teeth are intact, stable. There is no displacement of lower jaw fragments. X-ray picture shows a bone tissue defect up to 1 cm large. What prosthesis is indicated?
Question 140Select one:

A.
Oxman’s bridge-like prosthesis with pivot point

B.
Bridge-like prosthesis without a pivot point

C.
Lamellar prosthesis with Gavrilow’s pivot point

D.
Lamellar prosthesis with Oxman’s pivot point

E.
Clasp denture

A

A.
Oxman’s bridge-like prosthesis with pivot point

129
Q

An 18-year-old young man came to a dentist with complaints of sensitivity of his left lower teeth that is observed during eating sweet foods. Objectively, in the pericervical region on the vestibular surface of teeth 36 and 37, the enamel is matte white, without luster, fragile, chips off during probing. Make the clinical diagnosis.
Question 141Answer

a.
Chronic superficial caries

b.
Acute superficial caries

c.
Acute initial caries

d.
Chronic initial caries

e.
Chronic median caries

A

b.
Acute superficial caries

130
Q

During intraoral examination the dentist suspected that the patient has syphilis. What should the dentist do in this case?
Question 142Select one:

A.
Continue the examination and start the teeth preparation

B.
Decline to provide dental services for this patient

C.
Complete the examination and refer the patient for necessary tests

D.
Obtain the impressions to study the diagnostic models

E.
Inform the patient of the suspected diagnosis and end the visi

A

Complete the examination and refer the patient for necessary tests

131
Q

A 55-year-old man suffered a blow to the frontal mandibular area. He is diagnosed with mandibular fracture. Prior to trauma he was wearing removable dentures (partial laminar denture for the lower law and full denture for the upper jaw). What can be used for transport immobilization of the fracture?
Question 143Select one:

A.
Vankevych splint

B.
Vasiliev splint

C.
Zbarzh apparatus

D.
Patient’s dentures

E.
Weber splint

A

**
D.
Patient’s dentures**

131
Q

An 18-year-old girl needs consultation of the dentist regarding a neoplasm that appeared in the frontal area of her face. On the frontal area there is a pink-red semicircular spot on her skin 3-4 cm in size, with clear margins. When pressed, the skin in the affected area discolors and becomes pale. As soon as the pressing stops, the skin again assumes its pink-red color. When the patient bows her head, the spot darkens and enlarges in volume; when the patient lifts her head, the spot gradually returns to its initial pink-red color. Make the diagnosis:
Question 144Select one:

A.
Lymphangioma

B.
Neurofibromatosis

C.
Birthmark (nevus)

D.
Capillary hemangioma

E.
Cavernous hemangioma

A

D.
Capillary hemangioma

132
Q

A 60-year-old woman complains of a pain in the oral cavity that increases during eating. She has a history of ischemic heart disease. Objectively, her oral mucosa is pale pink, without visible pathological changes. On the lateral surface of the tongue there is an ulcer 0.7 cm in diameter. The ulcer has clear and even margins, is covered in a gray coating, and is painful on palpation. In the surrounding tissues there is no inflammatory reaction. Make the correct diagnosis:
Question 145Select one:

A.
Tuberculous ulcer

B.
Trophic ulcer

C.
Decubitus ulcer

D.
Syphilitic chancre

E.
Cancerous ulcer

A

B.
Trophic ulcer

133
Q

A 28-year-old patient complains of aesthetical defect. Objectively the crowns of 12, 11, 21, 22, and 23 are destroyed by caries by over 2/3 of their heights. The patient has orthognathic occlusion. X-ray shows the root canals of these teeth to be completely filled; no pathologic changes detected in the periapical tissues. What approach to teeth restoration would be optimal in this case?
Question 146Select one:

A.
Crown restoration with photopolymer fillings

B.
Stump inlays and porcelain-fused-to-metal crowns

C.
Combined swaged crowns, made in Borodiuk technique

D.
Immediate denture with posts

E.
Plastic-faced stump inlays

A

B.
Stump inlays and porcelain-fused-to-metal crowns

134
Q

A 55-year-old woman came to a dentist with complaints of burning in her oral cavity when eating irritant foods, a sensation of roughness, and an unusual appearance of the buccal mucosa. The patient has a history of chronic cholecystitis. Objectively, she has artificial crowns made of various metals in her oral cavity. Against the background of unchanged buccal mucosa, she has patches of small whitish papules in the retromolar space. They are keratinized, slightly raised, form a ring-shaped pattern, and cannot be removed by scraping. Make the provisional diagnosis.
Question 147Answer

a.
Acute pseudomembranous candidiasis

b.
Secondary syphilis

c.
Lichen ruber planus

d.
Lupus erythematosus

e.
Verrucous leukoplakia

A

c.
Lichen ruber planus

135
Q

A 12.5-year-old child came to the dentist for oral cavity sanitation. Objectively caries progression in this child is severe: decay-missing-filled index is DMF=2+0+4=6. Green-Vermillion index is 1.7. Gingival mucosa is pale pink. What toothpastes should be prescribed in this case for oral hygiene?
Question 148Select one:

A.
Amine fluoride

B.
Salt supplement

C.
Antiseptic

D.
Enzyme

E.
Herbal extract

A

A.
Amine fluoride

136
Q

A 24-year-old patient came to the dentist complaining of chalky lesions on the front teeth. Objectively teeth 11, 21, 31, 41 present with chalky lesions separated by areas of healthy unchanged enamel. Lesion surface is coarse; there is no response to thermal stimuli. Childhood years of the patient were spent in the area with fluorine level of 1.8 mg/L in drinking water. Make the diagnosis:
Question 149Select one:

A.
Endemic fluorosis

B.
Enamel hyperplasia

C.
Enamel hypoplasia

D.
Acute superficial caries

E.
Chronic superficial caries

A

A.
Endemic fluorosis

137
Q

A 13-year-old girl undergoes preventive examination. No complaints from the patient. Objectively: on the 36 masticatory surface there is a carious cavity within enamel with wide inlet. Enamel of the cavity floor and walls is dense and pigmented. Probing of the carious lesion is painless. No reaction to thermal stimuli. Tooth percussion is painless. The girl belongs to the I group according to her health status. Make the diagnosis:
Question 150Select one:

A.
Acute initial caries

B.
Chronic initial caries

C.
Chronic superficial caries

D.
Acute median caries

E.
Acute superficial carie

A

C.
Chronic superficial caries